解答

✨ 最佳解答 ✨

0≦θ<2π の各辺に π/4 を足すと π/4 ≦ t(= θ+π/4) < 9π/4

Wendy

理解出来ました!!ありがとうございます🙇‍♀️

留言
您的問題解決了嗎?

看了這個問題的人
也有瀏覽這些問題喔😉